Converging Sequences and Limits: Help with Induction Proof?

In summary, the conversation is about proving that the sequences an and bn converge, with the same limit, using induction. The induction hypothesis is that a_{m-1}<a_{m}<b_{m}<b_{m-1} and is used to show that a_{m+1}<a_m and b_{m+1}>b_m. To prove a_{m+1}<b_{m+1}, it is equivalent to showing that (a_{m+1})^2<(b_{m+1})^2, which can be done by manipulating the expressions and using the fact that a_i and b_i are positive.
  • #1
Swamifez
9
0
I've been working on this for the past hour, but haven't gone anywhere with it. If anyone can help to complete it, it would be highly appreciated. Thanks

Let 0< a1< b1 and define

an+1= √anbn

bn+1=(an+bn)/2a) Use induction to show that
an<an+1<bn+1<bn

Thus prove that an and bn converge.
b) Prove that they have the same limit.
 
Last edited:
Physics news on Phys.org
  • #2
1° show it's true for n=1 first.

2° assume it's true for n=m-1, i.e. assume [itex]a_{m-1}<a_{m}<b_{m}<b_{m-1}[/itex]

3° Use the part of the induction hypothesis that say [itex]a_{m}<b_{m}[/itex] to prove [itex]a_{m+1}<a_m[/itex] and [itex]b_{m+1}>b_m[/itex]. For the part [itex]a_{m+1}<b_{m+1}[/itex], notice that since the a_i and b_i are positive, it is equivalent to showing that [itex](a_{m+1})^2<(b_{m+1})^2[/itex], i.e. that [tex]a_nb_n<\frac{a_n^2+b_n^2}{4}+\frac{a_nb_n}{2}[/tex], etc. (think perfect square). That's enough hints. Go think for another hour. :smile:
 
  • #3


First of all, it is important to note that the given sequence is well-defined since an+1 and bn+1 are always positive (since √an and (an+bn)/2 are both positive for positive an and bn).

a) Base case: For n=1, we have a2=√a1b1 and b2=(a1+b1)/2. Since 0<a1<b1, we can see that a2<a1<b1<b2, satisfying the given inequality.

Inductive step: Assume that an<an+1<bn+1<bn for some arbitrary k. We will now prove that this holds for k+1 as well.

For an+1, we have:

an+1=√anbn<√bnbn=bn

For bn+1, we have:

bn+1=(an+bn)/2>(an+an)/2=an

Combining these two inequalities, we get:

an<an+1<bn+1<bn

Thus, by the principle of mathematical induction, we can conclude that an<an+1<bn+1<bn for all positive integers n. This also implies that the sequences an and bn are both increasing and decreasing respectively, hence they converge.

b) To prove that an and bn have the same limit, let L be the limit of both sequences. We will show that L satisfies the following equations:

L=√(L^2)=L/2

Solving these equations, we get L=0. Therefore, both sequences converge to 0, hence they have the same limit.

This completes the proof by induction.
 

1. What is an induction proof?

An induction proof is a mathematical method used to prove a statement or property for all natural numbers. It involves showing that the statement is true for the first natural number, and then showing that if it is true for any given number, it must also be true for the next number.

2. Why is induction used for proofs?

Induction is used for proofs because it allows us to prove a statement for an infinite amount of cases (all natural numbers) by only proving it for a few base cases. This saves time and effort compared to trying to prove each individual case.

3. How do I construct an induction proof?

To construct an induction proof, you must first show that the statement is true for the first natural number (usually 0 or 1). Then, assuming it is true for a given number, you must show that it is also true for the next number. This will prove that the statement is true for all natural numbers.

4. Can induction be used for any type of proof?

Induction can only be used for proofs involving natural numbers. It cannot be used for proofs involving real numbers or other types of mathematical objects.

5. What are some common mistakes to avoid in induction proofs?

Some common mistakes to avoid in induction proofs include assuming too much in the inductive step, not considering all cases, and not being specific enough in the base case. It is important to be clear and thorough in each step of the proof to ensure its validity.

Similar threads

  • Differential Geometry
Replies
1
Views
1K
Replies
18
Views
2K
  • Calculus
Replies
2
Views
1K
Replies
2
Views
841
Replies
2
Views
1K
Replies
3
Views
1K
Replies
2
Views
1K
Replies
2
Views
1K
Replies
4
Views
1K
Back
Top